【徹底解説】標本分散と不偏分散の分散

本記事は「これなら分かる!はじめての数理統計学」シリーズに含まれます。

不適切な内容があれば,記事下のコメント欄またはお問い合わせフォームよりご連絡下さい。

目次

標本分散と不偏分散の分散

平均$\mu$,分散$\sigma^{2}$の母集団からの標本分散を$S^{2}$,不偏分散を$U^{2}$とすると,

\begin{cases}
\displaystyle
V[S^{2}] = \frac{(n-1)^{2}}{n^{3}}\left\{\mu_{4}-\frac{n-3}{n-1}\sigma^{4}\right\}\\[0.7em]
\displaystyle
V[U^{2}] = \frac{1}{n}\left\{\mu_{4}-\frac{n-3}{n-1}\sigma^{4}\right\}
\end{cases}

が成り立つ。

証明

$Y_{i}=X_{i}-\mu$とおきます。$V[S^{2}]=E[S^{4}]-E[S^{2}]^{2}$を利用するため,標本分散の定義を確認します。

\begin{align}
E[S^{2}] &= \frac{n-1}{n}\sigma^{2}
\end{align}

次に,$E[S^{4}]$を計算します。

\begin{align}
E[S^{4}]
&= \frac{1}{n^{2}}E\left[\left(\sum_{i=1}^{n}(Y_{i}-\barY)^{2}\right)^{2}\right]\\[0.7em]
&= \frac{1}{n^{2}}E\left[\left(\sum_{i=1}^{n}Y_{i}^{2}-2\barY\sum_{i=1}^{n}Y_{i}+n\barY^{2}\right)^{2}\right]\\[0.7em]
&= \frac{1}{n^{2}}E\left[\left(\sum_{i=1}^{n}Y_{i}^{2}-n\barY^{2}\right)^{2}\right]\\[0.7em]
&=
\frac{1}{n^{2}}E\left[\left(\sum_{i=1}^{n}Y_{i}^{2}\right)^{2}\right]
-\frac{2}{n}E\left[\left(\sum_{i=1}^{n}Y_{i}^{2}\barY^{2}\right)^{2}\right]
+E\left[\left(\sum_{i=1}^{n}\barY^{4}\right)\right]
\end{align}

第一項目に関して,$i=j$となる項が$n$個,$i\neq j$となる項が$n(n-1)$個出現するため,$V[Y_{i}^{4}]=\mu_{4}$と$V[Y_{i}^{2}]=\sigma^{2}$に注意すると,

\begin{align}
\frac{1}{n^{2}}E\left[\left(\sum_{i=1}^{n}Y_{i}^{2}\right)^{2}\right]
&= \frac{1}{n^{2}}\left\{n\mu_{4}+n(n-1)\sigma^{4}\right\}
= \frac{1}{n}\left\{\mu_{4}+(n-1)\sigma^{4}\right\}
\end{align}

となります。第二項目に関して,$\barY^{2}=(\sum_{i} Y_{i})^{2}/n^{2}$となり,$E[Y_{i}]=0$に注意すると,第一項目と同様に$i=j$となる項が$n$個,$i\neq j$となる項が$n(n-1)$個だけが非ゼロの値として残るため,

\begin{align}
\frac{2}{n}E\left[\left(\sum_{i=1}^{n}Y_{i}^{2}\barY^{2}\right)^{2}\right]
&= \frac{2}{n^{3}}\left(n\mu_{4}+n(n-1)\sigma^{4}\right)
= \frac{2}{n^{2}}\left\{\mu_{4}+(n-1)\sigma^{4}\right\}
\end{align}

となります。第三項目に関して,第二項目と同様に$E[Y_{i}]=0$に注意すると,$i=j=k=l$のうち添え字が全て同じ項と,$2$つずつペアになる項だけが非ゼロの値として残ります。前者は$n$個の選び方があります。後者は$2$つずつのペアを決めるには片方を一意に決めればよく,$(i,j),(i,k),(i,l)$の$3$個の選び方があります。また,それぞれの添え字について$n(n-1)$個の番号の選び方があります。したがって,後者の項は$3n(n-1)$個出現します。

後者に関して,例えば$(j,k)$もペアの一つではないかと思われる方もおられるでしょう。$(j,k)$は$(i,l)$のペアを選んだ時点で自動的に定められているペアですので,選び方に加えてしまうと重複して数えていることになります。

したがって,第三項目は

\begin{align}
E\left[\left(\sum_{i=1}^{n}\barY^{4}\right)\right]
&= \frac{1}{n^{4}}\left\{n\mu_{4}+3n(n-1)\sigma^{4}\right\}
= \frac{1}{n^{3}}\left\{\mu_{4}+3(n-1)\sigma^{4}\right\}
\end{align}

となります。以上より,

\begin{align}
E[S^{4}]
&= \frac{1}{n}\left\{\mu_{4}+(n-1)\sigma^{4}\right\}-\frac{2}{n^{2}}\left\{\mu_{4}+(n-1)\sigma^{4}\right\}+\frac{1}{n^{3}}\left\{\mu_{4}+3(n-1)\sigma^{4}\right\}\\[0.7em]
&= \frac{n^{2}-2n+1}{n^{3}}\mu_{4}+\frac{n-1}{n^{3}}(n^{2}-2n+3)\sigma^{4}\\[0.7em]
&= \frac{(n-1)^{2}}{n^{3}}\mu_{4}+\frac{n-1}{n^{3}}(n^{2}-2n+3)\sigma^{4}
\end{align}

となります。したがって,

\begin{align}
V[S^{2}]
&= E[S^{4}]-E[S^{2}]^{2}\\[0.7em]
&= \frac{(n-1)^{2}}{n^{3}}\mu_{4}+\frac{n-1}{n^{3}}(n^{2}-2n+3)\sigma^{4}-\frac{(n-1)^{2}}{n^{2}}\sigma^{4}\\[0.7em]
&= \frac{(n-1)^{2}}{n^{3}}\mu_{4}+\frac{n-1}{n^{3}}(n^{2}-2n+3-n(n-1))\sigma^{4}\\[0.7em]
&= \frac{(n-1)^{2}}{n^{3}}\mu_{4}-\frac{n-1}{n^{3}}(n-3)\sigma^{4}\\[0.7em]
&= \frac{(n-1)^{2}}{n^{3}}\left\{\mu_{4}-\frac{n-3}{n-1}\sigma^{4}\right\}
\end{align}

となります。$V[U^{2}]$も同様にして導出できます。差分は$E[U^{4}]$で$1/(n{-}1)^{2}$が括り出される部分と$E[U^{2}]{=}\sigma^{2}$となる部分のみですが,不偏分散と標本分散の関係より$U^{2}{=}n/(n{-}1)S^{2}$であることを踏まえると,

\begin{align}
V[U^{2}]
&= \frac{n^{2}}{(n-1)^{2}}V[S^{2}] = \frac{1}{n}\left\{\mu_{4}-\frac{n-3}{n-1}\sigma^{4}\right\}
\end{align}

となります。

$V[U^{2}]$を求めてから$V[S^{2}]$を求めてもよいです。

参考文献

本稿の執筆にあたり参考にした文献は,以下でリストアップしております。

シェアはこちらからお願いします!

コメント

コメントする

※ Please enter your comments in Japanese to distinguish from spam.

目次